免費論壇 繁體 | 簡體
Sclub交友聊天~加入聊天室當版主
分享
返回列表 发帖

$(\prod_{i=1}^n(1+x_i))^{1/n} \ge 1+(\prod_{i=1}^nx_i)^{1/n}$

本帖最后由 abababa 于 2020-2-28 07:42 编辑

如题,如果$x_i>0$,是不是有
\[\left(\prod_{i=1}^{n}(1+x_i)\right)^{\frac{1}{n}} \ge 1+\left(\prod_{i=1}^{n}x_i\right)^{\frac{1}{n}}\]
感觉像是个著名的不等式,有没有什么名字?以及证明方法。
分享到: QQ空间QQ空间 腾讯微博腾讯微博 腾讯朋友腾讯朋友

一般地,卡尔松(Carlson)不等式了解一下
进一步地,holder 不等式了解一下

TOP

本帖最后由 abababa 于 2020-2-28 07:51 编辑

回复 2# 色k

holder 不等式到是听说过,还有个闵可夫斯基不等式,但是我都不会用。主楼那个,虽然提示了卡尔松不等式,但还是不知道要怎么证明。不行啊,我不等式太弱了。
先把卡尔松不等式的形式抄上:$a_{ij}\ge 0$,其中$i$从$1$到$n$,$j$从$1$到$m$
\[\left[\prod_{j=1}^{m}\left[\frac{1}{n}\sum_{i=1}^{n}a_{ij}\right]\right]^{\frac{1}{m}} \ge \frac{1}{n}\sum_{i=1}^{n}\left[\prod_{j=1}^{m}a_{ij}\right]^{\frac{1}{m}}\]

TOP

回复 3# abababa

突然想起,我在《数学空间》2013 年第 4 期(总第 14 期)的最后一页里写过 Carlson 不等式的均值证明……

TOP

本帖最后由 abababa 于 2020-2-29 13:50 编辑

回复 4# kuing

唉,最后也没用不等式的方式证明出来,用了数学归纳法:
当$n = 1$时显然成立,假设当$n = k$时成立,即
\[\left(\prod_{i=1}^{k}(1+x_i)\right)^{\frac{1}{k}} \ge 1+\left(\prod_{i=1}^{k}x_i\right)^{\frac{1}{k}}\]
令$D = \prod_{i=1}^{k}(1+x_i), M = \prod_{i=1}^{k}x_i, x_{k+1} = c > 0$,于是有$D^{\frac{1}{k}} \ge 1+M^{\frac{1}{k}}$,当$n = k+1$时,需要证明
\[\left(\prod_{i=1}^{k+1}(1+x_i)\right)^{\frac{1}{k+1}} \ge 1+\left(\prod_{i=1}^{k+1}x_i\right)^{\frac{1}{k+1}}\]
即要证明
\[\bigl((1+c)D\bigr)^{\frac{1}{k+1}} \ge 1+(cM)^{\frac{1}{k+1}}\]
即证明
\[(1+c)^{\frac{1}{k+1}} \cdot (D^{\frac{1}{k}})^{\frac{k}{k+1}} \ge 1+(cM)^{\frac{1}{k+1}}\]
而由归纳假设知$D^{\frac{1}{k}} \ge 1+M^{\frac{1}{k}}$,因此只要证明
\[(1+c)^{\frac{1}{k+1}} \cdot (1+M^{\frac{1}{k}})^{\frac{k}{k+1}} \ge 1+(cM)^{\frac{1}{k+1}}\]

因为$(1+M^{\frac{1}{k}})^{\frac{k}{k+1}} \ge 1+M^{\frac{1}{k+1}}$,所以只要证明
\[(1+c)^{\frac{1}{k+1}} \cdot(1+M^{\frac{1}{k+1}}) \ge 1+(cM)^{\frac{1}{k+1}}\]

即证明
\[(1+c)^{\frac{1}{k+1}}+\bigl((1+c)M\bigr)^{\frac{1}{k+1}} \ge 1+(cM)^{\frac{1}{k+1}}\]
这显然成立。

TOP

回复 5# abababa

《憋间》里的均值方法看不懂吗?

套用在这里的话,就是:把左边除到右边变成
\[\sqrt[n]{\frac{x_1x_2\cdots x_n}{(1+x_1)(1+x_2)\cdots(1+x_n)}}+\sqrt[n]{\frac1{(1+x_1)(1+x_2)\cdots(1+x_n)}}\leqslant1,\]然后由均值
\begin{align*}
\sqrt[n]{\frac{x_1x_2\cdots x_n}{(1+x_1)(1+x_2)\cdots(1+x_n)}}&\leqslant\frac1n\left(\frac{x_1}{1+x_1}+\frac{x_2}{1+x_2}+\cdots+\frac{x_n}{1+x_n}\right),\\
\sqrt[n]{\frac1{(1+x_1)(1+x_2)\cdots(1+x_n)}}&\leqslant\frac1n\left(\frac1{1+x_1}+\frac1{1+x_2}+\cdots+\frac1{1+x_n}\right),
\end{align*}相加即得证。

看懂的话,一般情况也就能理解了。而 holder 就是更一般的指数而已。

TOP

本帖最后由 abababa 于 2020-2-29 19:21 编辑

回复 6# 色k

谢谢,原题这个证明是看懂了,但我自己肯定想不出第一步两边除过去,这对我来说都太有技巧了。
人教论坛也上不去了,那个数学空间原来还看过一些,但对高考、教学的那些题兴趣不大,不等式一类的又看不懂,后来就没再看了。

TOP

返回列表 回复 发帖